1
$\begingroup$

Maybe it is too easy but I want to know that: If $R$ is regular local ring of Krull dimension $2$ and $m$ is the maximal ideal of $R$. (It means that height $m$ is $2$). Can we find any ideal of height $2$ different from $m$?

$\endgroup$
1
  • $\begingroup$ Rolled back a pointless edit of an off-topic question $\endgroup$
    – Yemon Choi
    Jul 26, 2014 at 13:27

1 Answer 1

2
$\begingroup$

No. Since it is the unique maximal ideal, $\mathfrak m$ contains every prime ideal (its complement is the set of all units of $R$).

$\endgroup$
0

Not the answer you're looking for? Browse other questions tagged or ask your own question.